Calculating the Delta of a Function with Zeros

Let's start from the beginning. What do we know about the Dirac Delta function and how does it behave?
  • #1
LocationX
147
0
I need to show that: [tex]\delta(g(x)) = \sum_k \frac{\delta(x-x_k)}{|g'(x_k)|} [/tex]

where the set [tex]{x_k}[/tex] are the zeros of g(x) and [tex]g'(x_k) \neq 0 [/tex]

I'm not really sure where to start for this problem, any clues would be much appreciated!
 
Physics news on Phys.org
  • #2
Hint: what is the defining property of the Dirac Delta function?
 
  • #3
Ok, so i have to show that [tex]\sum_k \frac{\delta(x-x_k)}{|g'(x_k)|}=0[/tex] for [tex]x \neq x_k [/tex] which I think a trivial step i think...

The final step would be to show that [tex]\int_{-\inf}^{\inf} \delta(g(x)) dx = \int_{-\inf}^{\inf} \sum_k \frac{\delta(x-x_k)}{|g'(x_k)|} dx =1 [/tex] when [tex]x= x_k [/tex] which is what i think would the the hard step...
 
  • #4
LocationX said:
Ok, so i have to show that [tex]\sum_k \frac{\delta(x-x_k)}{|g'(x_k)|}=0[/tex] for [tex]x \neq x_k [/tex] which I think a trivial step i think...

Right.

The final step would be to show that [tex]\int_{-\inf}^{\inf} \delta(g(x)) dx = \int_{-\inf}^{\inf} \sum_k \frac{\delta(x-x_k)}{|g'(x_k)|} dx =1 [/tex] when [tex]x= x_k [/tex] which is what i think would the the hard step...

Not quite. If there are more than one [itex]x_k[/itex], when you integrate from [itex]-\infty[/itex] to [itex]\infty[/itex], you will enclose all of them, and each will contribute 1 to the integral:

[tex]\int_{-\infty}^{\infty} \delta(g(x)) dx = \int_{-\infty}^{\infty} \sum_k \frac{\delta(x-x_k)}{|g'(x_k)|} dx=\sum_{k}(1)[/tex]
 
  • #5
Oh, that makes sense. What I'm confused about now is that the denominator will be different (the g'(x) term) and the sum will not come out to one?
 
  • #6
Sorry, I was mistaken in my previous comment,

[tex]\int_{-\infty}^{\infty} \delta(g(x)) dx \neq \sum_{k}(1)[/tex]

for much the same reason that

[tex]\delta(kx)=\frac{1}{|k|}\delta(x)\neq\delta(x)[/tex]

If you have proven this property, you can use the same method to prove your problem statement...
 
  • #7
a) suppose there is only a finite number of x_k, k=1,2,...n
b) Since g'(x_k)<>0 then there is en interval I_k where g' is monotonous and so invertible
c)choose I_k so that [itex]I_k \bigcap I_j=\phi, \quad for \quad k \neq j[/itex]
d)let C be the complement of unions of I_k (k=1,2,...n)
e)Now [itex]\int_{-\infty}^\infty=\int_C + \sum_k \int_{I_k}[/itex], so that the first integral of second member vanishes since its argument<>0
f)Integrate each term of the sum changing the integration variable [itex]x=g^{-1}(u)[/itex]

I think that's it
 
  • #8
gabbagabbahey said:
Sorry, I was mistaken in my previous comment,

[tex]\int_{-\infty}^{\infty} \delta(g(x)) dx \neq \sum_{k}(1)[/tex]

for much the same reason that

[tex]\delta(kx)=\frac{1}{|k|}\delta(x)\neq\delta(x)[/tex]

If you have proven this property, you can use the same method to prove your problem statement...

I'm not sure i follow, I think what you had was correct, that is:

[tex]
\int_{-\infty}^{\infty} \delta(g(x)) dx = \sum_{k}(1)
[/tex]

I'm not sure how to apply your statement below [itex]\left( \delta(kx)=\frac{1}{|k|}\delta(x)\neq\delta(x) \right) [/itex]

I see that the constant k factors below the delta, but in the problem, we have a function g(x) instead of a constant. Am I thinking about this correctly?
 
  • #9
Let's look at [itex]\int_{-\infty}^{\infty} \delta(g(x)) dx[/itex]... First break the integral into a bunch of intervals; some that just barely enclose exactly one of the zeroes of g(x) and some that enclose no zeroes. Obviously, the intervals that enclose no zeroes will integrate to zero (because the delta function will be zero over the entire interval), leaving you with something like:

[tex]\int_{-\infty}^{\infty} \delta(g(x)) dx=\sum_{k}\int_{x_k-\epsilon}^{x_k+\epsilon} \delta(g(x)) dx[/tex]

For some adequately small (by adequate, i mean small enough that each interval encloses only one zero of g(x)) positive number [itex]\epsilon[/itex]

Now make the substitution [itex]u=g(x)[/itex]...what do you get?
 
  • #10
Ok this is starting to make sense... I get:

[tex] \int \frac{\delta(u) du}{g'(x)} [/tex]

Am I assuming that g'(x) -> |g'(x)|?
 
  • #11
Well, what about the limits of integration?
 
  • #12
[tex]
\int ^{g(x_k + \epsilon)} _{g(x_k - \epsilon)} \frac{\delta(u) du}{g'(x)}
[/tex]

something like that? So basically the evaluated integral is [tex]\frac{1}{g'(x_k)}[/tex]
 
Last edited:
  • #13
LocationX said:
[tex]
\int ^{g(x_k + \epsilon)} _{g(x_k - \epsilon)} \frac{\delta(u) du}{g'(x)}
[/tex]

something like that? So basically the evaluated integral is [tex]\frac{1}{g'(x_k)}[/tex]

Well, you know the expected result is

[tex] \int ^{g(x_k + \epsilon)} _{g(x_k - \epsilon)} \frac{\delta(u) du}{g'(x)}=\frac{1}{|g'(x_k)|}=\left\{\begin{array}{lr}\frac{1}{g'(x_k)} & ,g'(x_k)>0\\\frac{-1}{g'(x_k)} & ,g'(x_k)<0\end{array}\right.[/tex]

Right?

Is there anything in the integrand that suggests this result? If not, then you need to take a closer look at the limits of integration...

Hint: You are free to choose [itex]\epsilon[/itex] to be infinitesimally small, when you do so, [itex]g(x_k\pm\epsilon)\approx[/itex]____?
 
  • #14
[tex]
g(x_k\pm\epsilon)\approx 0
[/tex]

Honestly, I'm not really sure where to go with this. I was thinking that the delta would give a -1 for g'<0 but that's not the case. I'm now thinking this:

[itex]
u=g(x) \rightarrow x=g^{-1}(u)
[/itex]

[tex]

\int ^{g(x_k + \epsilon)} _{g(x_k - \epsilon)} \frac{\delta(u) du}{g'(x)} = \int ^{g(x_k + \epsilon)} _{g(x_k - \epsilon)} \frac{\delta(u) du}{g'(g^{-1}(u))}

[/tex]

make epsilon infinitesimally small...

[tex]

\int ^{0} _{0} \frac{\delta(u) du}{g'(g^{-1}(u))} = \frac{\delta(0)}{g'(g^{-1}(0))} = \frac{1}{g'(x_k)}

[/tex]

I'm lost?
 
  • #15
LocationX said:
[tex]
g(x_k\pm\epsilon)\approx 0
[/tex]

Honestly, I'm not really sure where to go with this. I was thinking that the delta would give a -1 for g'<0 but that's not the case. I'm now thinking this:

I think you can make a little better approximation than that...perhaps a Taylor series?:wink:
 
  • #16
I get:

[tex]g(x) \approx g(a)+g'(a)(x-a)[/tex]
expand around x_k
[tex]g(x_k - \epsilon ) \approx g(x_k)+g'(x_k)((x_k - \epsilon ) - x_k)^2 [/tex]

[tex]

\int ^{g'(x_k)\epison ^2} _{g'(x_k)\epison ^2} \frac{\delta(u) du}{g'(x)}

[/tex]

?? Still a bit lost (sorry..)
 
Last edited:
  • #17
LocationX said:
I get:

[tex]g(x) \approx g(a)+g'(a)(x-a)[/tex]
expand around x_k
[tex]g(x_k - \epsilon ) \approx g(x_k)+g'(x_k)((x_k - \epsilon ) - x_k)^2 [/tex]

[tex]

\int ^{g'(x_k)\epison ^2} _{g'(x_k)\epison ^2} \frac{\delta(u) du}{g'(x)}

[/tex]

?? Still a bit lost (sorry..)

Surely you must mean:

[tex]g(x_k \pm \epsilon ) \approx g(x_k)+g'(x_k)((x_k \pm \epsilon ) - x_k)=\pm g'(x_k)\epsilon [/tex]

right?:wink:

This leaves you with:

[tex]\int ^{g'(x_k)\epsilon} _{-g'(x_k)\epsilon } \frac{\delta(u) du}{g'(x)}[/tex]

Now, what happens if g'(x_k) is negative...are you still integrating from a negative number to a positive number? If not, what happens to the sign of the integral?
 
  • #18
I get it now...

if g'(x_k)>0 then this integral still holds true:
[tex]
\int ^{g'(x_k)\epsilon} _{-g'(x_k)\epsilon } \frac{\delta(u) du}{g'(x)}
[/tex]

if g'(x_k)<0, then this integral becomes:
[tex]
\int ^{-g'(x_k)\epsilon} _{g'(x_k)\epsilon } \frac{\delta(u) du}{g'(x)}=-\int ^{g'(x_k)\epsilon} _{-g'(x_k)\epsilon } \frac{\delta(u) du}{g'(x)}
[/tex]

Thus the integral will still come out to be positive -g'(x_k) where g'(x_k)<0
 
  • #19
Right!:approve:
 

1. What is the purpose of calculating the delta of a function with zeros?

The delta of a function with zeros is used to determine the change in the output of a function when the input is changed by a specific amount. It is a measure of the sensitivity of the function to changes in the input.

2. How is the delta of a function with zeros calculated?

The delta of a function with zeros is calculated by finding the difference between the output of the function at two different input values, divided by the difference in the input values. This can be expressed as (f(x+h) - f(x)) / h, where h is the change in the input values.

3. What does a positive or negative delta indicate?

A positive delta indicates that the output of the function increases as the input increases, while a negative delta indicates that the output decreases as the input increases. This information is useful in understanding the behavior of the function and its rate of change.

4. Can the delta of a function with zeros be used to find the exact value of the function at a specific input?

No, the delta of a function with zeros only provides information about the change in output for a given change in input. It cannot be used to find the exact value of the function at a specific input, unless the initial value of the function at that input is known.

5. How does the number of zeros of a function affect its delta?

The number of zeros of a function does not directly affect its delta. However, the location of the zeros can impact the value of the delta, as the input values chosen for the calculation may be affected by the presence of zeros in the function.

Similar threads

  • Advanced Physics Homework Help
Replies
1
Views
917
Replies
1
Views
804
Replies
2
Views
1K
  • Advanced Physics Homework Help
Replies
4
Views
459
  • Advanced Physics Homework Help
Replies
1
Views
1K
  • Advanced Physics Homework Help
Replies
5
Views
1K
  • Advanced Physics Homework Help
Replies
4
Views
1K
  • Advanced Physics Homework Help
Replies
1
Views
687
  • Advanced Physics Homework Help
Replies
5
Views
2K
  • Advanced Physics Homework Help
Replies
19
Views
941
Back
Top